QUIZ 2 Flashcards

1
Q

Which of the following would be the BEST position to place a patient with peripheral arterial disease?

Sitting, legs dangling over the table
Semi-Fowlers. pillow under knees
Trendelenburg, pillow under head
Supine, pillow under ankle

A

sitting, legs dangling over the table

How well did you know this?
1
Not at all
2
3
4
5
Perfectly
1
Q

A patient presents with complaints of bilateral lower extremity edema related to chronic venous insufficiency. Which of the following would be the BEST treatment for this patient?

Compression stockings
Dependent position
Aspirin
Heparin

A

compression stockings

How well did you know this?
1
Not at all
2
3
4
5
Perfectly
2
Q

A PTA reviews the medical record of a patient who has been diagnosed with left sided congestive heart failure. The PTA would like to implement an exercise program but is concerned about the patient’s exercise tolerance. Which of the following abnormalities is the MOST LIKELY reason for the patient’s restricted exercise tolerance?

Reduced lung volumes
Arterial oxygen desaturation
Insufficient stroke volume during systole
Excessive increase in blood pressure

A

insufficient stroke volume during systole

How well did you know this?
1
Not at all
2
3
4
5
Perfectly
3
Q

Which of the following vessels takes deoxygenated blood from the heart to the lungs

Pulmonary artery
Pulmonary vein
Aortic artery
Left anterior descending artery

A

pulmonary artery

How well did you know this?
1
Not at all
2
3
4
5
Perfectly
4
Q

A patient begins to complain of dizzyness after moving from supine to sitting in bed s/p a total knee arthroplasty. The therapist notes in the chart that the patient’s resting supine blood pressure was 120/80. Which of the following BP reading in sitting would justify a diagnosis of orthostatic hypotension?

99/85 mmHg
106/82 mmHg
110/81 mmHg
107/76 mmHg

A

99/85

How well did you know this?
1
Not at all
2
3
4
5
Perfectly
5
Q

Patient: 75 y.o. male retired postal worker

Setting: Acute hospital

Diagnosis: 5 days s/p myocardial infarction, treated conservatively, non-STEMI

Meds: Beta blockers

POC: bed mobility and ambulate to tolerance

While ambulating the patient down the hall, the patient complains of feeling mildly out of breath and fatigued. Which of the following tests or vital signs would be the BEST indication of the patient’ s level of exertion?

Borg RPE scale

Pulse oximeter

Heart rate

Blood pressure

A

BORG RPE scale

How well did you know this?
1
Not at all
2
3
4
5
Perfectly
6
Q

Characterized by decreasing the sympathetic nervous system and causing a decrease in heart rate as well as allowing artery dilation

A

beta blockers

How well did you know this?
1
Not at all
2
3
4
5
Perfectly
7
Q

Preventing the conversion of angiotensin I to angiotensin II, this drug category allows decrease in blood pressure which helps hypertension and chronic heart failure

A

ACE inhibitors

How well did you know this?
1
Not at all
2
3
4
5
Perfectly
8
Q

This drug class relieves chronic heart failure patients by allowing the excretion of water and sodium to reduce the workload of the heart. Typical brand names include Lasix and Hydrochlorothiazide

A

diuretics

How well did you know this?
1
Not at all
2
3
4
5
Perfectly
9
Q

This medicine is used to decrease the clotting time of blood to prevent blood clots

A

anticoagulants

How well did you know this?
1
Not at all
2
3
4
5
Perfectly
10
Q

A patient with chronic heart failure has a surgical procedure to place a temporary pump on the ventricle to assist with contraction of the heart. This procedure would be BEST called:

A left ventricular assistive device
A heart transplant
Angioplasty with stenting
Coronary artery bypass graft

A

a left ventricular assistive device

How well did you know this?
1
Not at all
2
3
4
5
Perfectly
11
Q

Which of the following would you expect to see with an EKG of a person who sustained a STEMI myocardial infarction?

ST elevation
ST depression
Delay in P wave
Widened QRS wave

A

ST elevation

How well did you know this?
1
Not at all
2
3
4
5
Perfectly
12
Q

A 60-year-old male presents with sudden onset of shortness of breath, sharp chest pain that worsens with deep breaths, and a recent history of leg swelling, redness and warmth. Which condition is MOST likely?

Pneumonia
Pulmonary Embolism (PE)
Myocardial Infarction (MI)
Asthma

A

Pulmonary Embolism (PE)

How well did you know this?
1
Not at all
2
3
4
5
Perfectly
13
Q

a heart beat of more than 100 BPM

A

tachycardia

How well did you know this?
1
Not at all
2
3
4
5
Perfectly
14
Q

Which of the following is considered the “widow maker” due to its function of supplying blood to the left ventricle?

Left anterior descending artery
Pulmonary artery
Aortic artery
Right descending artery

A

left anterior descending artery

How well did you know this?
1
Not at all
2
3
4
5
Perfectly
15
Q

Which of the following is the semilunar valve that separates the left ventricle from the peripheral arterial system?

Aortic valve
Tricuspid
Bicuspid
Pulmonary

A

aortic valve

16
Q

This procedure involved rerouting the coronary arteries from a donor vessel such as the saphenous vein in order to improve circulation to the heart muscle.

Coronary artery bypass graft
Angioplasty
Stenting
Myoplasty

A

CABG - coronary artery bypass graft

17
Q

intermittent claudication is best described as

A

an arterial disease causing pain in legs with activity

18
Q

Patient: 65 y.o female w/ dyspnea at rest, fatigue with walking

Clinical symptoms: Bilateral lower extremity edema

Clinical history: COPD, decreased right ventricle funtion noted with ultrasound testing

Labs: Complete blood count: normal for RBCs and WBCs, negative for CPK/Troponin I/II

Based on the chart history given, the patient most likely has a diagnosis of:

Cor pulmonale
Anemia
Myocardial infarction
Hodgkin’s leukemia

A

cor pulmonale

19
Q

What heart sound is typically heard at the left second intercostal space near the sternum?

S1, indicating mitral and tricuspid valve closure
S2, indicating aortic and pulmonic valve closure
S3, indicating rapid ventricular filling
S4, indicating atrial contraction

A

s2, indicating aortic and pulmonic valve closure

20
Q

Which of the following would you MOST expect to see in someone with right-sided congested heart failure?

Distended jugular vein
Dry cough
Aortic aneurysm
Anemia

A

distended jugular vein

21
Q

Which of the following blood pressures would reflect a Stage I hypertension classification according to the American Heart Association guidelines?

132/80
100/75
142/95
80/60

A

132/80

22
Q

Which of the following would be the BEST test to determine if a patient’s subjective complaints of leg swelling, redness, and pain are needing further diagnostic evaluation?

Well’s criteria
Homan’s sign
Borg CV RPE
Palpation

A

well’s criteria

23
Q

This amount of blood that leaves the left ventricle after a ventricular contraction compared to the amount of blood in the left ventricle prior to contraction would BEST be called ______________ and should be around _____________ percentage.

Ejection fracture, 55-75%
Cardiac output, 40-60%
Heart rate, 60-90%
Blood pressure, 60-80%

A

ejection fracture, 55-75%

24
Q

Which of the following valves prevents the backflow of blood into the right atrium during ventricular systole?

Aortic valve
Pulmonary valve
Mitral valve
Tricuspid valve

A

tricuspid valve

25
Q

A note in the medical record indicates that a patient was recently prescribed Lasix (furosemide). Which of the following conditions is MOST commonly associated with use of this medication?

Atrial flutter
Deep vein thrombosis
Hyperlipidemia
Chronic heart failure

A

chronic heart failure

26
Q

A patient is walking on the treadmill when he begins to experience chest pain. He puts his hand over his chest and slows down the speed. After a few seconds, the chest pain resolves and he continues his walk. Which of the following would BEST characterized his complaints of chest pain?

Stable angina
Unstable angina
Myocardial infarction
Congestive heart failure

A

stable angina

27
Q

An Echocardiogram reveals a signficant decrease in a patient’s anticipated cardiac output. An INCREASE in which of the following variables would MOST LIKELY contribute to this finding?

Stroke volume
Afterload
Preload
Heart rate

A

afterload